Find the coordinates of the midpoint of LB if L(8, 5) and B(-6, 2)

Answers

Answer 1

Answer:

The coordinates of the midpoint of LB are (1, 3.5)

Step-by-step explanation:

The rule of the midpoint of a segment its endpoints are (x1, y1) and (x2, y2) is [tex]M=(\frac{x1+x2}{2},\frac{y1+y2}{2})[/tex]

Let us use this rule to solve the question

∵ M is the midpoint of segment LB

∵ The coordinates of point L are (8, 5)

x1 = 8 and y1 = 5

∵ The coordinates of point B are (-6, 2)

x2 = -6 and y2 = 2

→ Substitute these values in the rule of the mid point above

∵ [tex]M=(\frac{8+-6}{2},\frac{5+2}{2})[/tex]

→ Remember (+)(-) = (-)

∴ [tex]M=(\frac{8-6}{2},\frac{7}{2})[/tex]

∴ [tex]M=(\frac{2}{2},\frac{7}{2})[/tex]

M = (1, 3.5)

The coordinates of the midpoint of LB are (1, 3.5)


Related Questions

The yield in bushels from a grove of orange trees is given by Y = x(600 − x), where x is the number of orange trees per acre. How many trees will maximize the yield?

Answers

Answer:

90000

Step-by-step explanation:

Y = x(600 - x)

Let's rearrange the equation so that we have

Y = x(-x + 600)

Y = -x² + 600

When you take a look at parabolas, we know that their maximum or minimum values are usually on the axis of symmetry. This axis of symmetry has an equation of

x = -b/2a,

In this question, the axis of symmetry is

x = -600 / -2

x = 300

This means that planting 300 trees will maximize your yield. If you substitute for x = 300 into the equation, the maximum yield will be

y = 300 (600 - 300)

y = 90000.

Answer: 300

Step-by-step explanation:

use the formula X=-b/2a

x=-600/-2

x=300

f(x) = x + 7, g(x) = x - 7
(a) (f + g)(x) =
(b) (f-g)(x)=
(c) (fg)(x)=
(d) (f/g)(x)=

Answers

a) 2x

b) 14

c) x^2 -49

d) -1

The toco toucan, the largest member of the toucan family, possesses the largest beak relative to body size of all birds. This exaggerated feature has received various interpretations, such as being a refined adaptation for feeding. However, the large surface area may also be an important mechanism for radiating heat (and hence cooling the bird) as outdoor temperature increases. Here are data for beak heat loss, as a percent of total body heat loss, at various temperatures in degrees Celsius:

Temperature (degrees Celsius) 15 16 17 18 19 20 21 22 23 24 25 26 27 28 29 30
Percent heat loss from beak 35 36 38 28 41 43 55 46 39 54 45 58 60 56 62 67
a. The equation of the least-squares regression line for predicting beak heat loss, as a percent of total body heat loss from all sources, from temperature is: _____ (Use decimal notation. Give your answer to four decimal places.)

b. Use the equation to predict (
±
0.01) beak heat loss, as a percent of total body heat loss from all sources, at a temperature of 25 degrees Celsius.

c. What percent (
±
0.01) of the variation in beak heat loss is explained by the straight-line relationship with temperature?

d. Find the correlation r (
±
0.001) between beak heat loss and temperature.

Answers

Answer:

Kindly check explanation

Step-by-step explanation:

Given the data:

Temperature (degrees Celsius) : 15 16 17 18 19 20 21 22 23 24 25 26 27 28 29 30

Percent heat loss from beak : 35 36 38 28 41 43 55 46 39 54 45 58 60 56 62 67

Using an online regression calculator ; the regression equation obtained is :

ŷ = 2.0927X + 0.6029

X = independent variable

Y = predicted variable

2.0927 = slope

0.6029 = intercept

B.) temperature = 25

ŷ = 2.0927(25) + 0.6029

= 52.9204

C.) the explained variance is the value of the coefficient of determination (R²) which is the square of the correlation Coefficient

0.8785² = 0.7718

D.) the correlation Coefficient r is 0.8785 using the Coefficient of regression calculator

ax+6=15 a is a negative, what must be true about x?

Answers

Answer:

x= 9/a

Step-by-step explanation:

Quadrilateral GHJK has vertices G(2, 3), H(8, 2), J(6, 8), and K(3, 6). It is transformed according to the rule T(–4, –5).

What are the coordinates of G”?

(–7, 3)
(–2, 2)
(–1, –7)
(2, –2)

Answers

Answer:

the transformation T(a, b) can be change the pre image to the final image by following the rule as we explain below

if A(x, y) is a pre image, then A ' ( x', y') is its image, and x' = a+x, y' = b + y, where because of T(a, b) transformation

in our case Quadrilateral GHJK has vertices G(2, 3), so the coordinates of G”

can be found with

T(-4, -5) applied to G(2, 3) ⇒ G' (-4 +2, -5+3)=G' (-2, -2)

T(-4, -5) applied to G' (-2, -2) ⇒ G"(-4-2, -5-2) =G"(-6, -7)

the coordinates of G” are (-6, -7)

Step-by-step explanation:

i hope it helps?

Answer:

its c on edg (-1, -7)

Step-by-step explanation:

Pls I need help ASAP

Answers

Answer:

the answer is 1/8

thank you

if you loved my answer then please like it and mark as brainliest

The angle measures of triangle QRS are given:

The measure of angle Q is (5x−15)°.
The measure of angle R is 75°.
The measure of angle S is (3x)°.
What is the value of x?

Answers

Answer:

x = 15°

Step-by-step explanation:

(5x-15) + 75 + 3x = 180

8x + 60 = 180

8x = 120

x = 15°

Consider the hexagon ABCDEF what is the value of X? What is the measure of angle A? what is the measure of angle BCG?

Answers

Answer:

x = 52.5

<A = 152.5°

<BCG = 45°

Step-by-step Explanation:

Sum of interior angles of n-sided polygon = (n - 2)180

Sum of interior angles of hexagon = (6 - 2)180 = 4(180) = 720°

Therefore, sum of all the interior angles in the given hexagon = 720.

Thus:

3x - 5 + 2x + 140 + 135 + 2x + 30 + x = 720

Solve for x. Add like terms.

8x + 300 = 720

Subtract 300 from both sides

8x = 720 - 300

8x = 420

Divide both sides by 8

8x/8 = 420/8

x = 52.5

<A = (3x - 5)

Plug in the value of x

<A = 3(52.5) - 5 = 152.5°

<BCG = 180 - (2x + 30) (angles on a straight line)

Plug in the value of x

<BCG = 180 - (2(52.5) + 30)

<BCG = 180 - (105 + 30)

<BCG = 180 - 135

<BCG = 45°

HURRY PLEASE HELP

What is the approximate distance from the origin to the point (−10, −4, 5)? Round to the nearest tenth. 4.4 units 10.4 units 11.9 units 19.0 units

Answers

The answer is 11.9 units

First you must set up the distance formula.

[tex]\sqrt[]{ (-10-0)^2+(-4-0)^2+(5-0)^2}[/tex]

Then, simplify like terms.

[tex]\sqrt{(-10)^2+(-4)^2+(5)^2}[/tex]

Next, simplify exponents.

[tex]\sqrt{100+16+25}[/tex]

Then, add like terms.

[tex]\sqrt{141}[/tex]

Finally, round to the nearest tenth.

11.9

(-8) X 2/3 i NEED the answer help me

Answers

-5.33333333333

I used a calulator can I get brainliest please

What’s the difference between m=change in y and change in x and m =y2-y1and x2-x1

Answers

it's the same thing

[tex]slope = \frac{Δy}{Δx} or \frac{change \: in \: y}{change \: in \: x} = \frac{y2 - y1}{x2 - x1} = \frac{rise}{run} [/tex]

Show that each conjecture is false by finding a counterexample.


1.For any integer n, n3 > 0.


2. Each angle in a right triangle has a different measure.


3.For many years in the U.S, each bank printed its own currency. The variety of different bills led to widespread counterfeiting. By the time of the Civil War, a significant fraction of the currency in circulation was counterfeit. If one soldier had 48 bills, 16 of which were counterfeit, and another soldier had 39 bills, 13 of which were counterfeit, make a conjecture about what fraction of bills were counterfeit at the time of the Civil War.

Answers

Answer:

1.   [tex]n: n^3 > 0[/tex] is not true for negative integers

2.  [tex]45\ and\ 45[/tex] show that all angles do not have different measure

3.  [tex]Fraction = \frac{1}{3}[/tex]

Step-by-step explanation:

Solving (1):

[tex]n: n^3 > 0[/tex]

Take

[tex]n = -1[/tex]

[tex]-1^3 = -1[/tex]

[tex]n: n^3 > 0[/tex] is not true for negative integers

Solving (2):

The angles of a right angled triangle are:

[tex]45, 45\ and\ 90[/tex]

[tex]45\ and\ 45[/tex] show that all angles do not have different measure

Solving (3):

When

[tex]Bills = 48[/tex]

[tex]Counterfeit = 16[/tex]

The fraction is calculated as thus:

[tex]Fraction = \frac{Counterfeit}{Bills}[/tex]

[tex]Fraction = \frac{16}{48}[/tex]

[tex]Fraction = \frac{1}{3}[/tex]

Similarly, when

[tex]Bills = 39[/tex]

[tex]Counterfeit = 13[/tex]

[tex]Fraction = \frac{Counterfeit}{Bills}[/tex]

[tex]Fraction = \frac{13}{39}[/tex]

[tex]Fraction = \frac{1}{3}[/tex]

HELP URGENT !!!!!!!!!!!

Answers

The answer is B. -14*1=-14 and -14+1=-13.
I believe the answer is B. Please correct me if I’m wrong! Good luck!

subtract 8 from the input to get the output
divided the input by 3 to get the output

Answers

Answer: that will be 5

Step-by-step explanation:

Answer:

6

Step-by-step explanation:

If a person walks 1/3 of a mile every 1/6 of an hour, what is their speed in miles per hour?

Answers

Answer:

2 mi per hour

Step-by-step explanation:

1/3*6=

6/3= 2

2 mi per hour

Answer:

2mph

Step-by-step explanation:

If a person walks 1/3 of a mile every 1/6 of an hour you divide to solve.

[tex]\frac{\frac{1}{3}}{\frac{1}{6}}[/tex] To solve you flip the bottom and multiply:

[tex]\frac{1}{3} *\frac{6}{1}[/tex] = [tex]\frac{6}{3} = 2[/tex]

The person is walking 2mph

Need help ASAP with my math

Answers

U need a calculator for that lol

Answer:  The answer is -2

Step-by-step explanation: I want points and trust me this answer is correct

ayer 100% A company sells cases of tomato juice. Each case contains 24 cans of juice and sells for $18. The equation y = 18x represents the total price ( y), in dollars, when x number of cases are sold. Choose all statements about the situation that are correct based on the information. A. The cost per can is dependent on the number of cases sold. B. The number of cans sold is dependent on the price per can. C. The total price for a sale is dependent on the number of cases sold. D. The number of cases sold is independent from the number of cans in each case.​

Answers

Answer:

c

Step-by-step explanation:

Answer:

C is the answer

Solve for x for the following pls!
1) 7+3x+1=4x+5
2) 8+2x+18=x+19
3) 6+2x+13=x+26
would appreciate it sm

Answers

Answer:

1.x=3

2. x= -7    

3. x=7

Step-by-step explanation:

BI
Challenge 2
A: Can you find the
area of the circle?
r=x +1
B: Can you find the
circumference of
the circle?

Answers

Answer:

Area of the circle = pi * r^2 = pi * (x+1) ^ 2

Circumference of the circle = 2* pi * r = 2*pi*(x+1)

Plz do fast I need it for a test The front view of a tent is.shown below if a zipper.is going to be put in along the length x how long should the.zipper be

Answers

Answer:

x=15ft

Step-by-step explanation:

a^2+b^2=c^2

8^2+b^2=17^2

b^2+64=289

b^2=225

b=15

Evaluate 8 ÷ -2 · 4²+9. Thanks! Will give brainiest if possible

Answers

the answer is -55 , because you have to simply the 14^2 to 16
For more anwsers for questions like these use math away love even thought the ads might be irritating it’s a great source ✨

x+12x-13 = 0 and azo, what
is the value x+lo?​

Answers

Wat this doesn’t make sense

11. Factor for

12 – 8.1 + 16​

Answers

I’m going to go ahead and say the answer is 19.9... sorry if it’s wrong.

Answer:

2(6-4.05+8)

Step-by-step explanation:

2(6-48+43.05+8)

Can someone please help me I need help and I can’t understand this plsssss

Answers

1a. Multiply a variable by 13, add 2.

1b. Subtract two from both sides, divide both sides by 13. X=3

2a. Subtract 3 from a variable; divide by 5

2a. Multiply both sides by 5,add 3 to each side. x=-2
Solution (1) :

Solution (a) :

step 1. Multiply the variable x with 13 .

step 2. Add 2 to it .

Solution (b) :

Step 1 : Subtract 2 from both sides : 13x + 2 - 2 = 41 - 2 = 39

Step 2 : Divide both the sides by 13 : 13x ÷ 13 = 39 ÷ 13 = 3

x = 3

Solution (2) :

Solution (a) :

Step 1. Subtract 3 from the variable .

Step 2. Divide the result by 5 .

Solution (b) :

[tex]Step \: 1. \: add\: 3 \: to \: both \: the \: sides : \frac{x - 3}{5} + 3 = - 1 + 3

[/tex]

Step 2 : Multiply both the sides by 5 : 5 × 1/5x = 5 × -2/5

x = -2

The correct Standard Form of the equation y=−2/7x−11/7

Answers

Answer:

y=-(2)/(7)x-5

Step-by-step explanation:

How do you find what p is?

Answers

Answer:

p > -3

Step-by-step explanation:

This is an alternating p-series.  It is absolutely convergent if│an│converges.  1 / nᵖ⁺³ converges if the exponent is greater than 1.

p + 3 > 1

p > -2

The series is conditionally convergent if │an│diverges.  1 / nᵖ⁺³ diverges if the exponent is greater than 0 and less than or equal to 1.

0 < p + 3 ≤ 1

-3 < p ≤ -2

Combining these two, we get p > -3.

CONSTRUCTED-RESPONSE ITEM 14. Four people each deliver food to people's homes.
19
Constructed Responses
..
Curtis charges a flat fee of $2.50 for each delivery plus $0.20 per mile for each mile he drives. For one delivery, Curtis drives 6 miles.
...
A. How much does Curtis charge to deliver the food?

Answers

Answer:

I'm Pretty Sure it is $16.20

Step-by-step explanation:

First, you add 2.50 + 0.20 and you get 2.70. Then, you get 2.70 x 6 miles and you get 16.20. Hope this helps! :)

Salena needs to purchase 9 skirts for a new job. her friend, Jamie goes with her each of the skirts costs $41. Salena says that she can find the total cost by multiplying 9 and 4 and adding that to product of 9 and 1. Jamie disagrees and says that she needs to find the product of 9 and 40 and add that to the product of 9 and 1. Who is correct and why?

Answers

Answer:

Jamie is correct. This is because 41 is a combination of 40 and 1. Salena thinks that the 4 is just a 4, however, the 4 is in the tens place, so the 41 without the 1 is 40. The answer to this 41 times 9 is 369. Not 45.

Step-by-step explanation:

Each skirt is $41

Salena needs 9

(9*40) + (9*1)

360   +   9

    $369

Megan had $235.17 in her bank account. She withdrew $77.98 from the account to spend
on clothing. How much money does she have left in her bank account now?

Answers

Answer: 157.19

Step-by-step explanation:

157.19 that would be the answer

Simplest form of 8/36

Answers

Answer:

8.36 I think

Step-by-step explanation:

Answer: 2/9

Steps: 8/36 simplified is 2/9.

Plz mark brainliest:)

Other Questions
One of the dog treats in Shelleys shop is 8 inches long and 3.2 inches wide. She wants to make a miniature version that has proportional dimensions. If the miniature treat will be 3 inches long, how wide should it be? What is the meaning of this excerpt?Reread this excerpt from The Call of the Wild.An oath from Perrault, the resounding impact of a clubupon a bony frame, and a shrill yelp of pain, heralded thebreaking forth of pandemonium. The camp wassuddenly discovered to be alive with skulking furryforms,-starving huskies, four or five score of them,who had scented the camp from some Indian village.They had crept in while Buck and Spitz were fighting,and when the two men sprang among them with stoutclubs they showed their teeth and fought back. Theywere crazed by the smell of the food. Perrault found onewith head buried in the grub-box.O Perrault is fighting stray dogs that have invaded thecampsite looking for food.O Perrault is punishing members of his dogsled teamfor their aggressive acts.Perrault is reprimanding his dogsled team for stealingscraps of food.O Perrault is feeding wild dogs and training them forwork on his team. (4.3 x 10^-3)(2 x 10^-2) in standard form How did World War I change Americans attitude toward immigration PLEASEEEEEEQu palabra describe mejor el proceso de independencia de Mxico? An aquarium is being filled at a rate of 2.5 inches per second. The equationLaTeX: y=2.5xy = 2.5 x is used to determine the height of the water after LaTeX: xx seconds. Find the domain and range of the situation if it takes 60 seconds to fill the aquarium. The zika virus was first identified in Uganda in 1947. The virus is transmitted from person to person by a bite from a mosquito. It is commonly carried by the Aedes mosquito which is prevalent throughout Central Africa. The virus has more recently spread to the Americas (South, Central, and now North) and is expected to spread to Europe. There is a concern that the virus may reach epidemic proportions. Zika virus is related to yellow and dengue fever. Common symptoms are low grade fever and a distinctive skin rash and other symptoms include vomiting, headache, and muscle and/or joint pain. Pregnant women should avoid traveling to areas known to have zika virus present because the virus is known to cause microcephaly and developmental difficulties related to the central nervous system. Based on the paragraph above, class discussions, and other VALID (be careful with your selection of e-materials! CDC Center for Disease Control is a good place to start) sources of information please answer the following questions. 1. State a reason why the virus has spread to the Americas and likely to spread to Europe. 2. Define the term microcephaly.3. What bones of the fetal skull would be affected by zika? Specifically, what area of the CNS is not fully developed in microcephalic individuals. Which best explains why companies use celebrities to endorse their products? (pls helppp) HELP ME PLEASE IM ALMOST TO 100 For how many two-digit positive integers does the product of the digits come out to 12? Why was the Stamp Act a cause of the American Revolution? Quickkkk!!!!!!!! Help A hospital administrator emailed an agenda to all the staff employees for an upcoming meeting. The agenda listed thepoints below.-Discuss the change in policy for dealing with patient privacy rights.-Discuss the possible solutions to overcrowding in the waiting rooms.-Discuss possible fundraising ideas for the purchase of a spectrophotometer.What part of the meeting is being effectively communicated to staff members?the timeliness of the meetingthe objectives of the meetingthe meeting minutesthe administrator's negotiation skills Carlos sold 6 cellphones every hour during the day, but in the morning he already ha $1,250 in the register. What is the expression that represents the total amount of money that he should have in the register in a given hour?A: $1250B: 6xC: 6x + 1250D: 1250x + 6 Can yall help me please:) What is force and motion if there are 7 fish and 3 drown how many are left? what was the cause of the original political parties to form? Carta and Daniel bought identical suitcases at different stores. Carla's suitcase originally cost $75 and was discounted 19% Daniel's suitcase original cost sa5 and was on sale for 20% off the original price. Which suitcase was the bra? A Carta because her sutase cot $11 25 2. Daniel because his suitcase cot $17. Can cause decor, ST12 D. Dariel because ts sutcace was docoumed by S17 Choose the correct written expression for: (9 x 4) divided by 6.Options:he quotient of 9 and 4 times 6divided by the product of 9 and 4 the product of 9 and 4 divided by 69 times 4 divided by 6